If =x/y=5/8
then x/5=

Answers

Answer 1

[tex]\dfrac{x}{y}=\dfrac{5}{8}\Big|\cdot\dfrac{y}{5}\\\\\dfrac{x}{5}=\dfrac{y}{8}[/tex]

Answer 2

Answer:

x/5 = y/8

Step-by-step explanation:

x          5

----- = -------

y          8

Using cross products

8x = 5y

Divide each side by 8

x = 5/8 y

Divide each side by 5

x/5 = y/8


Related Questions

4+2p=10 (3/4p-2) solve for p

Answers

Answer:

p = 48/11 or 4.36

Step-by-step explanation:

4 + 2p = 10(3/4p - 2)

distribute the 10 on the right side of the equation

4 + 2p = (15/2p - 20)

multiply both sides by 2

8 + 4p = 15p - 40

move the terms

48 = 11p

p = 48/11

(sorry if this question is already answered, brainly is glitching out for me)

Answer:

p=6

I got it right on Kahn Academy

What is the approximate value of 2 -3

Answers

Assuming you are doing x minus y, the value would be -1.00

The number line can help - go left three times from 2 and you get an answer.

Assuming you are doing x times (-y), the value would be -6.00

Add -3 to itself 2 times

Please help with the question!!!!!

Answers

Answer:  [tex]y = -|x| + 1[/tex]

Start with the parent function [tex]y = |x|[/tex] which is a V shape graph that has its vertex at (0,0) and it is the lowest point. Every point has a positive y coordinate other than the vertex of course.

Reflect over the y axis to get [tex]y = -|x|[/tex], so now everything is negative or below the x axis, other than the vertex point.

Lastly, shift everything up 1 unit by adding 1 onto the right side getting [tex]y = -|x|+1[/tex]

Answer:

y=absolute value brackets -x end absolute value brackets +1

Step-by-step explanation:

this is the equation that matches

use an algebraic equation to find the measures of the two angles described below. begin by letting x represent the degree measure of the angles supplement.

the measure of the angle is twenty-nine times greater than its supplement

Answers

X+29x= 180
30x= 180
One angle is 6 the other is 174

PLS HELP!! Consider the exponential functions f, g, and h, defined as shown. Determine which function or functions have each key feature. Drag the tiles to the correct boxes. Not all tiles will be used.

Answers

Answer:

1) increases on all interval of x = only g(x)

2) approaches an integer as x approaches -∞ = only f(x)

3) y-intercept at (0, 4) = only g(x)

4) x-intercept at (1, 0) = f(x) and h(x)

Step-by-step explanation:

1) The functions that increases on all interval of x are;

f(x) = -2(3)ˣ + 6 decreases on all interval of x

The function g(x) which is 4 × 4ˣ  increases on all interval of x

The function h(x) is observed to be decreasing as x increases, therefore, the only function that increases on all interval of x = g(x)

2) The function that approaches an integer as x approaches -∞ = -2(3)ˣ + 6

-2(3)ˣ + 6 approaches 6 as x approaches -∞ which is only f(x)

3) The functions that have a y-intercept at (0, 4) is only g(x)

4) The function that have a x-intercept at (1, 0) are f(x) and h(x).

Answer:

A) f(x) and g(x)

B) only g(x)

C) All three functions

D) f(x) and h(x)

Step-by-step explanation:

In my uploaded image on the test I have gotten C) wrong but if you graph all three functions, you can see that all three functions have the same style of -∞

end behavior so I think that would be the most logical answer.

Journal Write a subtraction problem involving
numbers. At least one decimal must include hundredths. Explain how to solve
your problem.
Please help I will give you Brainlyest

Answers

Answer:

Problem: 5.24 - 4.93

Step-by-step explanation:

You subtract just like a normal Addition problem however you MUST make sure the decimal point line up,...

[tex]5.24\\4.93\\[/tex]

1st: subtract the 3 from the 4 [0.01]

2nd: subtract the 9 from the 2 (make the 2 a 12 (by carrying a one from the 5 (which then becomes a 4))) [0.30]

3rd: Finally subtract the 4 from the 4,... 0

Answer = 0.31,...

A more summed up answer: Subtracting (and adding for that matter) decimals happens exactly like normal. However you MUST MAKE SURE that your decimal points are lined up,... if you are subtracting a 0.09 from a 4.5 and the 4.5 doesn't have a hundredths place just remember that you can add a 0 on the end (4.50) without chancing it's value. (and then just subtract like normal)

7 people out of the 99 visitors bought a gift. About ___% of the visitors bought a gift.

Answers

Answer:

About 7.07% of the visitors bought a gift.

Step-by-step explanation:

7/99 = 0.0707

0.0707 *100 = 7.07%

then:

About 7.07% of the visitors bought a gift.

If a^b= b^a and a = 2b then find the value of a^2+b^2
a) 20
b) 30
c) 28
d) 24

Answers

Answer:

[tex]\large \boxed{\sf \bf \ \ a^2+b^2=20 \ \ }[/tex]

Step-by-step explanation:

Hello, please consider the following.

[tex]a^b=b^a\\\\\text{ We can replace a by 2b.}\\\\(2b)^b=2^b\cdot b^b=b^{2a}=b^2\cdot b^b\\\\\text{ Let's assume that b is different from 0 and we can divide by }b^b \\ \\ \text{ both sides of the equation, and it come.}\\\\2^b=b^2\\\\\text{ We find b = 2 and then a = 4, So}\\\\a^2+b^2=4^2+2^2=16+4=20[/tex]

Hope this helps.

Do not hesitate if you need further explanation.

Thank you

8c + 2(6 - 3c) - 7
How do I simplify this pls help

Answers

Answer:

2c+5

Step-by-step explanation:

Answer:

7c

Step-by-step explanation:

You do PEMDAS.

6-3c is 3c.

so itd be 8c + 2(3c) - 7

then id multiply 3c with 2 so its be6c.

then id add 6c with 8c, and its be 14c. then i subtract it with 7 and get 7c.

The circumference of a circle is 20π. What is the area of the circle?

Answers

Answer:

The area of the circle is 100 square units.

Step-by-step explanation:

We are given that the circumference of a circle is 20π, and we want to determine its area.

Recall that the circumference of a circle is given by the formula:

[tex]\displaystyle C = 2\pi r[/tex]

Substitute:

[tex]20 \pi = 2 \pi r[/tex]

Solve for the radius:

[tex]\displaystyle r = \frac{20\pi}{2\pi} = 10[/tex]

The area of a circle is given by:

[tex]\displaystyle A = \pi r^2[/tex]

Since the radius is 10 units:

[tex]\displaystyle A = \pi (10)^2[/tex]

Evaluate:

[tex]\displaystyle A = 100\pi\text{ units}^2[/tex]

In conclusion, the area of the circle is 100 square units.

PLS HELP ME
Given that the universal set.
U = {X : 30 ≤ X ≤ 40. X is an even number}.
P = {X : X is a multiple of 4}
and Q = {X : X is a number such that the sum of its digits is an odd number}.
find n(P∩Q')

A. 1
B. 2
C. 3
D. None of the above​

Answers

Given that

U = {X : 30 ≤ X ≤ 40. X is an even number}.

so Mathematically

U = {30,32,34,36,38 40}.

P = {X : X is a multiple of 4}

P = { 4,8,12,16,20,24,28,32,36,40,44,........}

and Q = {X : X is a number such that the sum of its digits is an odd number}.

Q = {12,14,16,18,21,23,25,27,29,30,32,34,36,38,41,43,45,..........}.

(Q')= U-Q  ={40}

(P∩Q')= {40}

n(P∩Q')= 1

(Q') is Q complement which means a set containing those elements of the universal set which are not element of the set Q.

(P∩Q') is a set containing those elements of set P and Q complement which are common to both sets.

n(P∩Q') denotes the number of elements in a set containing those elements of set P and Q complement which are common to both sets which  has only 1 element which is 40.

So the best answer is part A = 1.

https://brainly.com/question/11439790

https://brainly.com/question/13729460

Special right triangles

Answers

Answer: please find the attached files

Step-by-step explanation:

A unit circle formula and special triangle of 45, 30 and 60 degrees can be used to solve the problem.

Please find the attached files for the solution

Can someone answer this please? Okay so it says that something is made 3 times than the other item. The other item uses 13 beads. So, what is 13 times 3?

Answers

Answer:

13x3= 39

Step-by-step explanation:

10x3=30

3x3=9

30+9=39

Hope it helps!

If you multiply them it would be 39. Hope this helps

Rational and irrational numbers are both imaginary numbers.
True
False
help please! thank you :)

Answers

Answer:

The correct option is;

False

Step-by-step explanation:

An imaginary number is also known as a imaginary part of a complex number is a real number that has a factor of √(-1)

Rational numbers are numbers that can be put in the form of the ratio of two integers (real numbers), forming a simple fraction such as 1/2, or 3/7

Irrational numbers are the subset of real numbers that cannot be expressed as a ratio of two numbers such as π, √2, Eulers number, e, the golden ratio, φ

Therefore, rational numbers and irrational numbers are real numbers not imaginary numbers.

4x − 1 < 11 solve for x

Answers

Answer:

[tex]\huge \boxed{x < 3}[/tex]

Step-by-step explanation:

4x - 1 < 11

Add 1 on both sides.

4x - 1 + 1 < 11 + 1

4x < 12

Divide both sides by 4.

(4x)/4 < 12/4

x < 3

6p- 2q for p = 3 and q = -7

Answers

Answer:

32

Step-by-step explanation:

plug in the values of the variables then simplify

Answer:

[tex]\boxed{32}[/tex]

Step-by-step explanation:

Hey there!

Well given the expression,

6p - 2q

and p and q we need to plug them in.

6(3) - 2(-7)

Simplify

18 - -14

18 + 14

= 32

Hope this helps :)

Mr.Snyder gave his four children $35 to split equally for each car they cleaned out. The children cleaned put 3 cars. Mr.Snyder does not have any coins. He only had dollar bills. How much money should each child get?

Answers

Answer:

$26 for each child

Step-by-step explanation:

35 * 3 = 105 / 4 = 26.25

i just answered this, they would each get $26


16. Mrs. Acosta wants to buy 2 flag pins
for each of the 168 band members for
the Fourth of July Parade. Pins cost
$0.09 each. Which is the best estimate
of the cost of the pins?


Answers

Because we are looking for an estimate, we can round 168 people to 170. Then, 2 pins for about 170 people would be 340 pins (simply multiple 170 x 2). Then we can round 9 cents to 10 cents, or .1 dollars. 340 pins times .1 is just 34 dollars (move the decimal once to the left).

Answer: About 34 dollars

ans this question......​

Answers

Step-by-step explanation:

Universal Set(U)={2,3,4...16}

M={2,4,6,8..16}[Even numbers between 2-16]

N={3,5,7...15}[Odd numbers between 2-16]

(PUM)'=U-(PUM)={2,3,4...16}-[{2,4,6,8..16}U{3,5,7...15}]={}

For shading, Shade the area outside the two circles but inside the square.

14 POINTER IF YOU HIT THE BULLSEYE!!!!! HELP!
Billy, Bob, and Joe found a question in their test that said this and they NEED HELP!!!!! with flying promises to give brainliest they now wait.......

Answers

Answer:

mean would decrease

Step-by-step explanation:

PLS HELP! Given the angles in the diagram below, what is m<2

Answers

Your question answer is d 98 is right.

I own a large truck, and my neighbor owns four small trucks that are all identical. My truck can carry a load of at least 600 pounds more than each of her trucks, but no more than 1/3 of the total load her four trucks combined can carry. Based on these facts, what is the greatest load I can be sure that my large truck can carry, in pounds?

Answers

Answer:

The load the large truck can carry  is 2400 pounds

Step-by-step explanation:

Let the load the large truck can carry  = X

Let the  load each of the four trucks owned by the neighbor can carry = Y

The given parameters are;

The load the large truck can carry X = 600 + Y......(1)

The number trucks owned by the neighbor = 4

The load the large truck can carry X ≤ 1/3 × 4 × Y

Therefore, X ≤ 4/3 × Y

At maximum capacity, we have;

X = 4/3 × Y

Substituting the value of X into equation (1), we have;

4/3 × Y = 600 + Y

600 = 4/3 × Y - Y = 1/3·Y

Y = 3 × 300 = 1800 pounds

Y =  = 1800 pounds

Therefore, the load the neighbors truck can carry = 1800 pounds

X = 600 + Y gives;

X = 600 + 1800 = 2400 pounds

∴ The load the large truck can carry  = 2400 pounds

Cole deposits $500 into his savings account. The bank pays him 2% interest on the amount he has in his account. What expression would you

use to find the total Cole has in his account?

Answers

Answer:

500*2%=10

500+10=510

Step-by-step explanation:

i need help. Can u help me solve for x?

Answers

Answer:

[tex] x = \sqrt {40}[/tex]

Step-by-step explanation:

Given is an isosceles triangle, dotted line is the bisector of top angle which is also perpendicular bisector of the base of the triangle. Hence, by Pythagoras theorem:

[tex] {x}^{2} = {6}^{2} + ({ \frac{4}{2} })^{2} \\ = 36 + 4 \\ = 40 \\ \therefore \: x = \sqrt{40} \\ [/tex]

Answer:

D. x = sqrt(52).

Step-by-step explanation:

Since the line measuring 6 units bisects the top angle, there are two right angles. We can use the Pythagorean Theorem to solve for x.

a^2 + b^2 = x^2

4^2 + 6^2 = x^2

16 + 36 = x^2

52 = x^2

x = sqrt(52)

x = sqrt(2 * 2 * 13)

x = 2sqrt(13)

x = 7.211102551.

Hope this helps!

Please answer this question now

Answers

Answer:

320 square inches

Step-by-step explanation:

4 * 1/2(8)(16) + 8*8 = 320

Answer:

320 sq. in.

Step-by-step explanation:

The formula for finding the area of a triangle is:

[tex]\frac{hb}{2}[/tex] (basically multiplying the height and the base and then dividing by 2)

Since there are 4 triangles, we can multiply the area of 1 triangle by 4 (64 times 4 is 256).

Then, on the bottom we have a (8 times 8) square (64).

Triangles: 256

Square: 64

256 + 64 = 320 sq. in!

Hope that helps and maybe earns a brainliest!

Have a great day!

The number of fish in the lake can be modeled by exponential regression equation y equals 14.08 * 2.08 X where X represents the year which is the best prediction for the number of fish in your 6 round your answer to the nearest whole number

Answers

Answer:

1140

Step-by-step explanation:

The best prediction for the number of fish in year 6 is 1517.

What is regression?

Regression is a statistical method used to analyze the relationship between two or more variables.

It helps to identify and quantify the relationship between the dependent variable (also called the response variable) and one or more independent variables (also called the explanatory variables or predictors).

We have,

To find the best prediction for the number of fish in year 6, we need to substitute x = 6 into the exponential regression equation:

So,

y = 14.08 x [tex]2.08^x[/tex]

y = 14.08 x [tex]2.08^6[/tex]

y = 14.08 x 107.6176

y = 1516.672768

Rounding to the nearest whole number, the best prediction for the number of fish in year 6 is 1517.

Thus,

The best prediction for the number of fish in year 6 is 1517.

Learn more about regressions here:

https://brainly.com/question/28178214

#SPJ7

1.5(a -4) - 2 = 4 pls help ASAP and show checked answer

Answers

Answer:

4

Step-by-step explanation:

Solving:

1.5(a -4) - 2 = 4 1.5a - 6 -2 = 41.5a - 8 = 41.5a = 4 + 81.5a = 12a = 12/1.5a = 8

Checking:

1.5(8-4) - 2 = 41.5*4 - 2 = 46 - 2 = 44 = 4
Solution :
5a -20 - 2 = 4
5a - 22 = 4
5a = 4 + 22
5a = 26
a = 26/5
a = 5.2
Check :
5(5.2 - 4 ) - 2 = 4
26 - 20 - 2 = 4
6 - 2 = 4
4 = 4
EQUATION PROVEN ✅

Please help pleaseeee

Answers

Answer:

False

Step-by-step explanation:

When you compare the dots to the numbers, you can see that the 5s and 8s don't match.

The dot plot has 6 5s, but the data has 5.

The dot plot has 2 8s, but the data has 1.

I hope this helps!

pls ❤ and give brainliest pls

Organize the data set:

8,7,5,5,4,2,3,3,4,6,5,4,5,5

2,3,3,4,4,4,5,5,5,5,5,6,7,8

False

Find the sum and express it in simplest form.
(6u - 7c - 6) + (-2u + 4c) ​

Answers

Answer:

4u - 3c - 6

Step-by-step explanation:

Remove the parentheses and combine like factors.

(6u - 7c - 6) + (-2u + 4c)

6u - 7c - 6 - 2u + 4c

4u - 3c - 6

find the area of a sector with a central angle of 150 and a diameter of 7.8 cm

Answers

Answer:

19.9 cm²

Step-by-step explanation:

The following data were obtained from the question:

Angle at the centre (θ) = 150°

Diameter (d) = 7.8 cm

Area of sector =.?

Next, we shall determine the radius.

This can be obtained as illustrated below:

Radius (r) = Diameter (d) /2

r = d/2

Diameter (d) = 7.8 cm

Radius (r) =.?

r = d/2

r = 7.8/2

r = 3.9 cm

Finally, we shall determine the area of the sector as follow:

Angle at the centre (θ) = 150°

Radius (r) = 3.9 cm

Pi (π) = 3.14

Area of sector (A) =.?

A = θ/360 × πr²

A = 150/360 × 3.14 × 3.9²

A = 15/36 × 3.14 × 15.21

A = 19.9 cm²

Therefore, the area of the sector is 19.9 cm²

Other Questions
Explain why they substituted cos(60) with 1/2 ? (Look at image) When you roll two number cubes, what are the odds, in simplest form, in favor of getting two numbers less than 3? A. 1:8 B. 8:1 C. 1:9 D. 9:1 which objects would have a greater gravitational force between them, Objects A and B, or Objects B and C In a mathematics class, half of the students scored 86 on an achievement test. With the exception of a few students who scored 46, the remaining students scored 77. Which of the following statements is true about the distribution of scores Which of the following is composed of a population of a species plus populations of other species? A. Community B. Biotic Potential C. Abiotic community D. Ecosystem b. Does refactoring mean that you modify the entire design iteratively? If not, what does it mean? Economists study poverty and income inequality to answer which of the following questions?Select one:O a. How does labor-force experience affect wages?O b. What are people's wages?O C. How do people adjust their behavior due to taxation?O d. How much inequality is there in society? Solve for x.X + 102x - 50x = [?] Which substance has the highest boiling point?Select the correct answer below:A) ethyl etherB) ethyl alcoholC) waterD) these all have the same boiling point Which of the fractions below are less than 2/5? Select two. Please help ASAP!!!!!! In 1960, the Soviets withdrew from a planned summit after Group of answer choices their ambassador to Great Britain defected to the West. American protestors threw eggs at the motorcade of Premier Nikita Khrushchev. American trade restrictions prevented them from buying wheat. they shot down an American spy plane. Next Write a sentence with a subordinate clause functioning as a modifier to the subject. Write six hundred twelve thousand, three hundred in the place-value chart. Then write the number in expanded Can anyone give me definitions on these words 2- 3 sentences for each Sustainability, Water Scarcity, Water balance and Environmentally Friendly need help...!! plzzz HEEELLLPPP PLSS!!! Compare and contrast open, closed, and isolated systems. Be sure to discuss the exchange of energy and exchange of matter, and provide at least one example of each. a rectangular garden is 52m long and 34m broad a path 2m wide is running inside the garden calculate the cost of travelling the path at rs 40 per sq metre snd calculate the cost of covering the empty space with tires at rs 20 per sq metre a country region that has a separate geographic territory Oilers, Inc. refines and markets its energy products in different nations around the world. In addition, Oilers' stockholders and managers come from many different nations. If some of the nations where it operates decided to take over the assets of the company, this act would constitute